A bathtub that holds 32 gallons of water contains 12 gallons of water. begin filling it, and after 5 minutes the tub is filled. write a standard equation

Answers

Answer 1

Answer:

In standard equation format, the condition can be written as [tex]V=5x+12[/tex].

Step-by-step explanation:

The capacity of the bathtub is 32 gallons.

The water already in the tub is 12 gallon.

It takes 5 minutes for the tub to be filled.

Let x be the flow rate of water through tap, in gallon per minutes.

Now, the water filled in the tank in 5 minutes will be [tex]5x[/tex] gallons.

So, in equation, the given condition can be written as,

[tex]V=5x+12\\32=5x+12[/tex]

here V represents the volume of the bathtub.

Therefore, in standard equation format, the condition can be written as [tex]V=5x+12[/tex].

For more details, refer the link:

https://brainly.com/question/22527528?referrer=searchResults

Answer 2
Final answer:

The standard equation for the volume of water in the bathtub at time t is V(t) = 12 + 4t, indicating the tub starts with 12 gallons and fills at a rate of 4 gallons per minute.

Explanation:

To write a standard equation for filling a bathtub, we need to find the rate at which the tub fills. Since the tub starts with 12 gallons and ends up holding 32 gallons, it needs an additional 20 gallons to be filled. If it takes 5 minutes to fill these 20 gallons, then the fill rate is 20 gallons per 5 minutes.

The equation for the volume V of water in the tub at any given time t (in minutes) is:


V(t) = 12 + r × t

where r is the rate of filling the tub. We already calculated r to be 4 gallons per minute (20 gallons/5 minutes). Therefore, the equation becomes:


V(t) = 12 + 4t

This equation represents the amount of water in the bathtub at any given time t, assuming it starts with 12 gallons and fills at a rate of 4 gallons a minute.


Related Questions

The length to width ratio of a wide-screen tv is 16:9. if a tv is 30 inches long, how wide is it?

Answers

it is about 16.9 inches wide
around range its about 16.2 

Ian wants to promote his band on the internet. Site a offers website hosting for $4.95 per month with a $49.99 startup fee. Site b offers website hosting for $9.95 per month with no startup fee. Write and solve an inequality to determine how many months lan could have his wedsite on site b and still keep his total cost less than site a

Answers

 I believe for twelve (12) months. 

Site A: $49.95 plus $59.40(4.95 x 12) equals $109.35 

Site B: $9.95 x 12 equals $119.40 

No, wait that's not right. 

Okay, at 8 months, site A is pretty much at $90 (forget the nickels) and site B is $80 so site B is less. 
At 7 months, site A is $85 and site B is $70 so site B is less. 
At 9 months, site A is $95 and site B is $90 so site B is less. 
At 10 months, site A $100 and site B is $100 

It's got to be around 10 months somewhere. 

Ten months would be $99.45 for site A and $99.50 for site B so B is less. 

Eleven months is $104.40 for A and $109.45 for B so now B is more.
Final answer:

Ian could have his website on site B for less than 10 months for it to be cheaper than hosting on site A.

Explanation:

To solve this question, we need to find when the costs between site a and site b equal each other. Since site a charges $4.95 per month plus a $49.99 startup fee, the cost of site a can be modelled by the equation: 4.95x + 49.99. Site b charges $9.95 per month with no startup fee, so its cost can be modelled by the equation 9.95x. We can now set up the inequality as follows:

4.95x + 49.99 > 9.95x

To solve for x, we will subtract 4.95x from both sides of the inequality, getting 49.99 > 5x. Finally, we divide each side by 5 to solve for x, resulting in x < 10. So Ian could have his website on site b for less than 10 months and still keep his total cost less than site a.

Learn more about Inequality Problem-Solving here:

https://brainly.com/question/15086723

#SPJ12

What is the equation for the linear model in the scatter plot obtained by choosing the two points closest to the line.
A. y= -1.5x + 6
B. y= 1.5x +6
C. y= 1.5x - 12
D y=2x+6

Answers

We can rule out choice A because the slope for choice A is negative, but the line is not moving downhill (read it from left to right)

We can also rule out choice C as the y intercept is definitely not -12. The y intercept is some positive value above 5, so 6 is probably a good guess or the correct y intercept.

This leaves choice B or choice D. They both have a y intercept of 6, so let's ignore that part. The slopes are different. For choice B we have 1.5 as the slope and choice D we have 2 as the slope.

Pick the two points closest to the line which are (4, 12) and (20,36). Start by using the graph to find the two closest points. Then use the table to confirm that you have the proper numeric values.

Slope formula:
m = (y2-y1)/(x2-x1)
m = (36-12)/(20-4)
m = 24/16
m = 1.5

So this points to choice B as the final answer

Option B is correct. The required for the linear model in the scatter plot is y = 1.5x + 6

The standard formula of expressing the equation of a line is;

y = mx + b where;

m is the slope

b is the y-intercept

Using the coordinate points (20,36) and (4, 12)

Get the slope:

[tex]Slope = \frac{12-36}{4-20}\\Slope = \frac{-24}{-16}\\Slope=\frac{3}2}=1.5[/tex]

The point where the line meets the y-axis is the t intercept. From the graph, the y-intercept is b = 6

Get the required equation:

[tex]y=mx+b\\y=1.5x + 6[/tex]

Hence the required for the linear model in the scatter plot is y = 1.5x + 6

Learn more here: https://brainly.com/question/17003809

the posted speed limit in many parts of europe is 100km/hr. what is the equivalent speed limit in miles per hour?

Answers

1 m = 1.609 km 1 km = 0.621371 miles .62 miles x 100 = 62 miles per hour

it is not d it is c i just took the test

Answers

ok so nevermind i won't help


Factor this trinomial completely. -6x3 + 26x2 + 20x   A. -2x(3x + 5)(x - 2)   B. -2x(3x + 2)(x - 5)   C. -2(3x + 2)(x - 5)   D. -2(3x + 5)(x - 2)

Answers

-6x3 + 26x2 + 20x
= -2x(3x^2 - 13x + 10)
= -2x (3x + 2)(x - 5)

answer
 B. -2x(3x + 2)(x - 5)

Perimeter of the rectangular playing field is 470 yards. the length of the field is 5 yards less than triple the width, what are the dimensions of the playing field

Answers

P = 2(L + W)
P = 470
L = 3W - 5

470 = 2(3W - 5 + W)
470 = 2(4W - 5)
470 = 8W - 10
470 + 10 = 8W
480 = 8W
480/8 = W
60 = W <==== width is 60 yards

L = 3W - 5
L = 3(60) - 5
L = 180 - 5
L = 175 <==== length is 175 yards

Kyle needed to find the solution to a system of equations by graphing. According to the graph, what is the solution?

A) (2,6)
B) no solution
C) imaginary numbers
D) infinite solutions
Kyle needed to find the solution to a system of equations by graphing. According to the graph, what is the solution?

A) (2,6)
B) no solution
C) imaginary numbers
D) infinite solutions

Answers

It appears that ur lines are coincident.....they lie on top of each other....therefore, they have infinite solutions

Please help with this question thank you!

Answers

x + 5 = 2x - 1
x = 6

6 + 5 = 11 or B

Sue spent $40 on one pair of shoes and earned $35 baking cookies. Write an expression using positive and/or negative integers to represent this situation. Evaluate the expression and interpret what it means.

Answers

She would be $5 in debt.

What is true about a pseudoscientific idea?

It can be replicated and verified.
It is improved with new information.
It ignores parts of the method within an investigation.
It tries to explain natural phenomena by analyzing, observing, and testing.

Answers

Pseudoscience is when the scientific method is either ignored or not applied correctly, which leads to wrong claims, data and similar issues.

so the solution is It ignores parts of the method within an investigation.


The answer is (C.) It ignores parts of the method within an investigation.

What is pseudoscience?

Pseudoscience is a proposition, a finding, or a system of explanation that is presented as science but that lacks the rigor essential to the scientific method. Pseudoscience can also be the result of research that is based on faulty premises, a flawed experimental design, or bad data.

Among the most notable developments in the history of pseudoscience in the 19th century is the rise of Spiritualism (traced in America to 1848), homeopathy (first formulated in 1796), and phrenology (developed around 1800).

Learn more about pseudoscientific here: https://brainly.com/question/604092

#SPJ2

.
A normal probability plot of the survival times of the guinea pigs in a medical experiment is shown below. Use this plot to describe the shape of the distribution of survival times. Then, explain carefully how this shape is seen in the normal probability plot. That is, make the connection between the normal probability plot and the shape of the data.

https://api.agilixbuzz.com/Resz/~hdAcBAAAAAQ99yZmb7l1SA.XDuW_wcASaP2dYyG_RjrDD/62689880,708,1,1,0/Assets/flvs/apstatistics_v9_gs-xml/res0019/01_16a_e.gif

Answers

the probability plot shows that the guinea pigs that survived increasingly grew quickly, most likely when they started getting all of the guinea pigs done some  came along and that is what the staggering points on the grid.

The graph of a proportional relationship contains the point (8, 4).

What is the corresponding equation?

Enter your answer as a fraction in simplest form

Answers

[tex]\bf \qquad \qquad \textit{direct proportional variation}\\\\ \textit{\underline{y} varies directly with \underline{x}}\qquad \qquad y=kx\impliedby \begin{array}{llll} k=constant\ of\\ \qquad variation \end{array}\\\\ -------------------------------\\\\ (\stackrel{x}{8}~~,~~\stackrel{y}{4})\implies \begin{cases} x=8\\ y=4 \end{cases}\implies 4=k8\implies \cfrac{4}{8}=k\implies \cfrac{1}{2}=k \\\\\\ \boxed{y=\cfrac{1}{2}x}[/tex]

what is 7,925 divided by 72?

Answers

110 is the answer to your question you can type it into a calculator to make sure
110.069444444 search it up on google and it'll tell you the answer on a calculator

Bananas cost $0.70 each, and peaches cost $0.90 each. If you want to spend exactly $21.20 at the stand and you need to buy 11 bananas, how many peaches can you buy? Write an equation in standard form modeling this situation then see how many peaches you can buy. Let be represent the number of bananas you buy and p represent the number of peaches you buy. [2 points]
Standard Form Equation: _______________________
p = ________

Answers

0.7b+0.9p = 21.2 is the equation.

You can buy 15 peaches, P = 15.

Plugging 11 in for b (equation above), you have spent $7.70 on bananas. Subtract this number from $21.20 to find that you have $13.50 left over for peaches. Divide 13.5 by 0.9 (since peaches cost 90 cents each), and your answer is 15.

With $21.20, you can buy 11 bananas and 15 peaches.

Mr. Rose spent $63 for a sport jacket and a pair of slacks. If the jacket cost $33 more than the slacks, how much did he pay for each?
Which system of equations represents the word problem if j is the jacket price and s is the price of the slacks?

j + s = 63 and s - j = 33
j + s = 63 and j - s = 33
js = 63 andj/s = 33

Answers

the equation would be j+s=total
we know that the total is 63, and the jacket cost s+33
it is addition and subtraction, so we can cancel the last one. j is more than s, so we would go with your second option, j + s = 63 and j - s = 33. 
Can I plz have brainliest?

Answer:

The system of the equations is: j+s=63 and j-s=33

The Jackets price is j=48

The slacks prices is s=15

Step-by-step explanation:

You know for the problem that:

j+s=65

And

j= s+33; from this equation you get j-s=33

Then you replace j "j=s+33" in the first equation:

(s+33)+s=63

2s+33=63

2s=63-33

s=(63-33)/2

s=15

and you know that: j=s+33

Then, j=15+33=48

and j+s=63

48+15=63

The domain of a radical function is the set of all x for which the radicand is positive. True or false?

Answers

Final answer:

The domain of a radical function is the set of all x for which the radicand is positive.

Explanation:

The statement in the question is almost correct. The domain of a radical function is the set of all x for which the radicand is non-negative, not just positive.

A radical function is defined as a function that contains a radical expression, which is the square root or any higher-order root of a variable or expression. The domain of a radical function is the set of all values of x that make the radicand, which is the expression inside the radical, positive.

For example, let's consider the function f(x) = √(x+3). The radicand x+3 can only be positive for values of x greater than -3. Therefore, the domain of this function is (-3, ∞).

Learn more about Radical function here:

https://brainly.com/question/31910587

#SPJ2

What's the answer and how do you do it?

Answers

the vertical asymptotes for a rational expression, occur at the values of "x" that make the denominator 0, and therefore the expression undefined.

what are those anyway?  simple, just set the denominator to 0, solve for "x", and that will spit them out.

x + 4 = 0
x = -4

there, if "x" ever becomes -4, then the fraction turns to [tex]\bf \cfrac{1}{x+4}\implies \cfrac{1}{-4+4}\implies \stackrel{und efined}{\cfrac{1}{0}}[/tex]

thus, that's where the vertical asymptote is at, x = -4.
A vertical asymptote is when y gets higher and higher or lower and lower without stopping around a certain point. In a fraction one way this can happen is when there is a point where the denominator is 0, as when it gets close to 0 it gets bigger and bigger, and anything over 0 is undefined. Therefore the answer is A as 1/4-4 is 1/0 and an asymptote


Solve the compound inequality shown below. Show steps.
-4< 3x -1<5

Answers

Add 1 to all parts.
-4+1<3x-1+1<5+1
-3x<3x<6
-3/3<3x/3<6/3
=-1hope this helps!

Can you show me the steps for this equation and answerF(x)=(x-3)^2+2

Answers

Let's simplify step-by-step.
(
x

3
)
2
+
2
Distribute:
=
x
2
+

6
x
+
9
+
2
Combine Like Terms:
=
x
2
+

6
x
+
9
+
2
=
(
x
2
)
+
(

6
x
)
+
(
9
+
2
)
=
x
2
+

6
x
+
11
To solve f(x) = (x-3)^2 +2:
1. Apply the quadratic formula
2. Remeber to expand the equation
You will get x^2-6x+11, and after applying the formula, you will get imaginary numbers. I don't know if this problem is right, sorry.

Apply Mathematics (1)(
a. A company’s weekly revenue R is given by the formula R = - p^2+ 30p, where p is the price of the company’s product. The company is considering hiring a distributor, which will cost the company 4p + 25 per week.

Answers

The values of price p for the product to remain profitable must be less than 1 and 25.

The value of P that will maximize the profit is 13.

Maximum function

For the given product to remain profitable, the revenue must continue to be greater than the cost that is R(p) > C(p)

Given the following parameters:

[tex]R(p) = - p^2+ 30p\\C(p)=4p+25[/tex]

If R(p) > C(p), then;

[tex]- p^2+ 30p>4p+25\\p^2-26p+25<0[/tex]

Factorize to get the value of p

[tex]p^2-p-25p+25<0\\p(p-1)-25(p-1)<0\\(p-1)(p-25)<0\\p<1 \ and \ p<25[/tex]

Hence the values of price p for the product to remain profitable must be less than 1 and 25

To maxmimize the profit, dP/dp = 0

Profit = Revenue - Cost

Profit = [tex]- p^2+ 30p - 4p - 25[/tex]

Profit = [tex]-p^2+26p-25[/tex]

Differentiate the function and equate to zero;

[tex]dP/dp = -2p+26\\-2p + 26 = 0\\-2p = -26\\p=13[/tex]

Hence the value of P that will maximize the profit is 13

Learn more on cost and revenue functions here: https://brainly.com/question/25638609

Spencer has a rectangular garden. The length and width are both odd numbers and the perimeter is 16 feet. What can be the area of his garden? Find all possible answers.

Answers

Area of rectangle = length x width

Perimeter of rectangle = 2 x (length + width)

Now we know that perimeter of rectangle = 16

2(length + width) = 16

Length + width = 16/2

Length + width = 8

Now the value of length and width is that if we add both values we have the answer 8.

So, here are some possible values of length and width and also that length and width are odd numbers;

Odd numbers are = 1,3,5,7,….

7 + 1 = 8

5 + 3 = 8

So, either length is 7 and with is 1 or length is 5 and width is 3

So area of rectangle = 7 x 1 = 7 or 5 x 3 = 15

Thus, the possible values of area of rectangle are 7 ft² and 15 ft².

 

Which is bigger, 978 centimeters or 12 meters?

Answers

12 meters because when you change 12 meters into centimeters (12*100), then it equals to 1200 centimeters.

how do you graph a function

Answers

Find the answer of the equation if it has one and mark those on the graph

Simplify (4x3+ 13x − 7) − (6x2+ 9x + 2). −2x2+ 4x − 9 −2x2+ 22x − 5 4x3− 6x2+ 4x − 9 4x3− 6x2+ 22x − 5

Answers

(4x³+ 13x − 7) − (6x²+ 9x + 2)
Remember that if they don't have the same exponents/bases (x²,x³, etc.), then they cannot be grouped together.

This means that 4x³ and 6x² will be left alone because there are no other x² or x³ in the expression.

So first thing you do is distribute. Don't forget about that - in front of the second parentheses.
(4x³+ 13x − 7) − (6x²+ 9x + 2)
(4x³ + 13x − 7) + (−6x² - 9x - 2)
4x³ - 6x² + (13x - 7) + (-9x - 2)
4x³ - 6x² + 4x (- 7) + (- 2)
4x³ - 6x² + 4x - 9

So the answer is:
C. 4x3− 6x2+ 4x − 9

List the next three terms in the following sequence: 1, 2, 4, 8, 16, 32,... a. 48, 80, 128 b. 64, 96, 160 c. 64, 128, 256 d. 46, 92, 184

Answers

C
because they are doubling


C because the numbers multiply by 2

The sum of negative eighteen and a number is eleven. What is the number?

Which equation could be used to solve the problem?

x - 18 = 11
18 - x = 11
-x + 18 = 11
x + 18 = 11

Answers

-18 +x = 11

rewrite as x-18 =11

x = 11+18 = 29

Answer:

x-18=11

step by step explanation

if your average stride length is 2.6 feet how many strides will it take you to walk to grandparents house

Answers

The distance from your house to your grandparents house is 3.67 miles or 19377.6 ft.

So the total number of strides needed would be:

strides needed = 19377.6 ft / (2.6 ft / stride)

strides needed = 7,452.92 = 7,453 strides

As children grow, dosages for medications gradually approach those for adults. Clark’s rule is often used to determine the correct dosage for children.

Child’s dose = Adult dose x (Weight of child in pounds)/(150 pounds)

What dosage should be administered to a child weighing 28 pounds if the adult dose of a certain drug is 80 milligrams? Round your answer to the nearest whole milligram.
How much would a child have to weigh to receive the adult dosage? Explain why your answer is reasonable.

Answers

A child weighing 28 pounds would have to take 15 milligrams of medicine in order to receive and adequate dose, if the adult dosage is 80 milligrams. For the child to receive the full adult dosage of 80 milligrams, the child would have to weigh 150 pounds.

A meteor crater is 3300 feet in diameter. Approximate the distance around the crater. Use 3.14 for pie.

Answers

circumference = diameter x PI

 3300 *3.14 = 10,362 feet

Answer:

Distance around the crater is 10362 feet.

Step-by-step explanation:

Given : A meteor crater is 3300 feet in diameter.

To find : Approximate the distance around the crater. Use 3.14 for pie.

Solution: We have given Diameter = 3300 feet.

Radius = [tex]\frac{Diameter}{2}[/tex].

Radius = [tex]\frac{3300}{2}[/tex].

Radius = 1650 feet.

Circumference = 2 * pi* radius.

Circumference = 2 *3.14* 1650.

Circumference = 3.14 * 3300.

Circumference = 10362 feet.

Therefore, Distance around the crater is 10362 feet.

Other Questions
Keys to preventing a fall from a ladder include choosing the right ladder for the job and tying the top and bottom of the ladder to fixed points. What else can you do to keep safe when using a ladder?A. Always keeping both hands on the ladderB. Keep your hands free of carrying tools or other materials when climbing a ladderC. Make sure you wear safety googlesD. Never climbing a ladder higher than 4 feet Solve for r: d = rt *A. d = rtB. t = d/rC. r = d/tD. r = t/d How might some native Americans in the southwest have felt about Spanish missions? What exact date did columbus discover america? Visual acuity in animals primarily depends on the _______. What force is needed to accelerate a 55kg shopping cart 2.0m/s to the second power? Julia plays volleyball and finished the regular season with 25 digs. The graph relates her total number of digs for the season to the number of games played in the post-season tournament. Graph in the coordinate plane with title Julias volleyball digs. Vertical axis goes from 0 to 60 in increments of 5 and is labeled number of digs. Horizontal axis goes from 0 to 10 in increments of 1 and is labeled games played. A ray on the graph starts at point begin ordered pair 0 comma 25 end ordered pair and goes through points begin ordered pair 4 comma 37 end ordered pair and begin ordered pair 8 comma 49 end ordered pair. What is the equation in standard form that represents the relationship shown in the graph? y=3x+25 y37=3(x4) 3x+y=25 3x+y=25 y=3x+25 The Hamptons family has a fish tank holding 10,450 ml of water.The water is leaking at a rate of 270 ml per minute.Part A: Define the input and output in the given scenario.Part B: Write a function to model this situation.Part C: How many liters of water will be left in the tank after 10 minutes.? suppose f(x)=x-2. describe the transformation from the graph of f(x) to the graph of g(x)=x+3. Use a graphing calculator to check your answer. If the interest rate on a savings account is 0.018% approximately how much money do you need to keep in this account for one year to earn enough interest to cover a single $9.99 below minimum balance fee A marathon is 26 miles 385 yards long . That is about what 1.4 x10 feet how many feet long is half a marathon Simplify:4p + 12 3q 4p 18 + 6qA.8p + 3q 6B.8p 3q + 6C.3q + 6D.3q 6 Recent studies have demonstrated that infantile amnesia can occur for __________ memories without affecting __________ memories for the same information. If the equatorial diameter of the sun is 1.39 million km how many kilometers is the sun's radius What point of view uses the pronouns he she and they and lets reader know the thoughts and emotions of all the storys characters What is 24 divided by 1272 The chain of custody is used for what purposes A trough is 12 ft long and its ends have the shape of isosceles triangles that are 5 ft across at the top and have a height of 1 ft. If the trough is being filled with water at a rate of 13 ft3/min, how fast is the water level rising when the water is 8 inches deep?I have been getting 13/30, but apparently, that is wrong?V=.5 * h * l * bBecause of similar triangles:b = 5h (b/h = 5/1)l = 12V = .5 * h * 5h * 12V = 30h^2(dV/dt) = 60 h (dh/dt)dV/dt = 13h = .5 (because this is in feet)13 = 60*.5(dh/dt)13/30 = dh/dt A decrease in supply, with no change in demand, will lead to ________ in equilibrium quantity and ________ in equilibrium price.a. an increase; an increaseb. an increase; a decreasec. a decrease; an increased. a decrease; a decrease What is the volume of a cube that measures 5 cm on each side? Steam Workshop Downloader